Deriving the General Lorentz Transformation | Special Relativity

แชร์
ฝัง
  • เผยแพร่เมื่อ 5 ก.ย. 2024

ความคิดเห็น • 22

  • @redandblue1013
    @redandblue1013 11 หลายเดือนก่อน +2

    Very nice, my course's lecture notes absolutely sucked and I needed some clarification on this. Finally found the full matrix

  • @lowersaxon
    @lowersaxon 11 หลายเดือนก่อน

    Well, thats correct of course. There are many ways to picture the LT and by applying the economics of Occam‘s razor I think we should use the easiest way to do.

  • @Abon963
    @Abon963 ปีที่แล้ว +1

    Thanks a lot!

  • @milessodejana2754
    @milessodejana2754 4 หลายเดือนก่อน

    Wait, in case of t --> t', if t = 0, does that mean that negative value of t' is valid?

  • @michael_zaki6903
    @michael_zaki6903 11 หลายเดือนก่อน +1

    Thank you this was the only video I could find involving the derivation of this specific matrix. You just helped me a ton.

    • @deepbean
      @deepbean  11 หลายเดือนก่อน

      Glad it was helpful!

  • @NH-zh8mp
    @NH-zh8mp 7 หลายเดือนก่อน

    This is a general Lorentz boost in arbitrary direction. What about the more general case where the boost is applied for an inertial frame (t''',z) whose 3 spatial axises are a rotation of of a initial frame (t,x) ? Is the time coordinate in this frame (t''',z) coincide with the time coordinate in the frame (t',y) which is a boost of (t,x) ?

  • @NH-zh8mp
    @NH-zh8mp 7 หลายเดือนก่อน

    what about using Lie algebra to find the form of the general Lorentz transformation ?

  • @sathyanarayananviswanathan3770
    @sathyanarayananviswanathan3770 ปีที่แล้ว

    great video, really enjoyed it

    • @deepbean
      @deepbean  ปีที่แล้ว

      Glad you enjoyed it!

  • @HomoLamarckiens
    @HomoLamarckiens 10 หลายเดือนก่อน +1

    Awesome video that I wanted for long

  • @NH-zh8mp
    @NH-zh8mp ปีที่แล้ว

    I still don't get the point at 1:39, may you explaine more detail on why you got that second term on the right in that formula ?

    • @deepbean
      @deepbean  ปีที่แล้ว +1

      Hi!
      So basically, the idea is that the current velocity vector can be separated into two components, one of which is parallel to the velocity, and one of which is perpendicular. The perpendicular component remains unchanged while the parallel component is modified according to the position Lorentz transform (derived in the video before this one in the series).

    • @NH-zh8mp
      @NH-zh8mp ปีที่แล้ว

      @@deepbean I got it. I still got some questions.
      (1) can you explaine more about the equality at 2:22 ?
      (2) how do you conclude that it is proj_(vec v) of vec r ?
      (3) how about the equaliy at 2:47 ?

    • @deepbean
      @deepbean  ปีที่แล้ว +1

      @H N Hi, apologies for the late reply. So this is just the idea that the position vector can be decomposed into components perpendicular to and parallel to the velocity vector, the latter being just the projection of the position vector on the velocity vector, which you can calculate using the dot product. That equation at 2:47 is just a way of finding the projection via the dot product. Hope this answers it.

  • @Benjamin-gx8ho
    @Benjamin-gx8ho 8 หลายเดือนก่อน

    around 2:16, why do you make the velocity have an arrow in the second equation?

    • @premanav
      @premanav 7 หลายเดือนก่อน

      Good Question. Hope we got the answer soon.

    • @premanav
      @premanav 7 หลายเดือนก่อน

      My guess is with an arrow on velocity make it free from parallel component in this context and direction of velocity vector is now is the direction of relative velocity direction of inertial frames in Lorentz transformation happening.

    • @deepbean
      @deepbean  4 หลายเดือนก่อน

      That's because it's now a vector. So, on the first line, we have a unit vector on the second RHS term, which we then get rid of by vectorizing the two terms in the bracket.

  • @NH-zh8mp
    @NH-zh8mp ปีที่แล้ว +1

    So, is there a linear algebra way to gain this general Lorentz transformation ?

    • @deepbean
      @deepbean  ปีที่แล้ว +2

      Yup! If you wanted, you could also do this by using rotation matrices; rotating the velocity vector such that it lies completely on the x-axis, then applying the 1-D Lorentz transform, then rotating back again.